If the student takes neither physics nor writing, then it could be true that the student also takes neither

Maria-Marin on April 17, 2020

Can you explain why C is wrong please?

Can you explain why C is wrong please?

Replies
Create a free account to read and take part in forum discussions.

Already have an account? log in

shunhe on April 17, 2020

Hi @Maria-Marin,

Thanks for the question! Let’s take a look at this question. We’re told that the student takes neither P nor W. This means that there are 5 courses left the student can choose from: H, L, M, S, and T.

Now, first, let’s take a look at (B). Let’s say the student doesn’t take H and M. Then the courses left in are L, S, and T. If we take a look at the rules, then none of them are violated, so (B) is fine and is the correct answer.

Now, let’s take a look at (C). Let’s say the student doesn’t take S and H. That means the student has to take L, M, and T, since we know that the student has to take at least 3 courses out of the 7. But now take a look at rule 2. If M is taken, then T cannot be taken. Since (C) violates rule 2, it can’t be the correct answer.

Hope this helps. Feel free to ask any further questions that you might have.

on June 5 at 03:19PM

The tutor makes this video moves entirely too fast and hardly explains his logic. This is consistent across all of the videos he is in. By far they worst LSAT Max tutor.